Fields finitely generated as $mathbb Z$-algebras are finite?












14












$begingroup$


Suppose $k$ is a field that is finitely generated as a ${mathbb Z}$-algebra. (That is, $k$ is a quotient of ${mathbb Z}[X_1,dots,X_n]$ for some $n$). Does it follow that $k$ is finite?










share|cite|improve this question











$endgroup$








  • 2




    $begingroup$
    A quick glance at another post makes me think you should be able to first show that $K$ is of prime characteristic. Perhaps once that's established, you can determine the degree of $K$ over its prime subfield is finite?
    $endgroup$
    – vgty6h7uij
    May 23 '12 at 13:04


















14












$begingroup$


Suppose $k$ is a field that is finitely generated as a ${mathbb Z}$-algebra. (That is, $k$ is a quotient of ${mathbb Z}[X_1,dots,X_n]$ for some $n$). Does it follow that $k$ is finite?










share|cite|improve this question











$endgroup$








  • 2




    $begingroup$
    A quick glance at another post makes me think you should be able to first show that $K$ is of prime characteristic. Perhaps once that's established, you can determine the degree of $K$ over its prime subfield is finite?
    $endgroup$
    – vgty6h7uij
    May 23 '12 at 13:04
















14












14








14


15



$begingroup$


Suppose $k$ is a field that is finitely generated as a ${mathbb Z}$-algebra. (That is, $k$ is a quotient of ${mathbb Z}[X_1,dots,X_n]$ for some $n$). Does it follow that $k$ is finite?










share|cite|improve this question











$endgroup$




Suppose $k$ is a field that is finitely generated as a ${mathbb Z}$-algebra. (That is, $k$ is a quotient of ${mathbb Z}[X_1,dots,X_n]$ for some $n$). Does it follow that $k$ is finite?







commutative-algebra finite-fields






share|cite|improve this question















share|cite|improve this question













share|cite|improve this question




share|cite|improve this question








edited Nov 1 '14 at 18:03









user26857

39.3k124183




39.3k124183










asked May 23 '12 at 12:52









WillOWillO

2,3321321




2,3321321








  • 2




    $begingroup$
    A quick glance at another post makes me think you should be able to first show that $K$ is of prime characteristic. Perhaps once that's established, you can determine the degree of $K$ over its prime subfield is finite?
    $endgroup$
    – vgty6h7uij
    May 23 '12 at 13:04
















  • 2




    $begingroup$
    A quick glance at another post makes me think you should be able to first show that $K$ is of prime characteristic. Perhaps once that's established, you can determine the degree of $K$ over its prime subfield is finite?
    $endgroup$
    – vgty6h7uij
    May 23 '12 at 13:04










2




2




$begingroup$
A quick glance at another post makes me think you should be able to first show that $K$ is of prime characteristic. Perhaps once that's established, you can determine the degree of $K$ over its prime subfield is finite?
$endgroup$
– vgty6h7uij
May 23 '12 at 13:04






$begingroup$
A quick glance at another post makes me think you should be able to first show that $K$ is of prime characteristic. Perhaps once that's established, you can determine the degree of $K$ over its prime subfield is finite?
$endgroup$
– vgty6h7uij
May 23 '12 at 13:04












4 Answers
4






active

oldest

votes


















18












$begingroup$

Yes!

Consider the morphism
$f:mathbb Zto k$ and the ideal $mathfrak m=f^{-1}(0)subset mathbb Z$.

Since $mathbb Z$ is a Jacobson ring and $(0)subset k$ is maximal, $mathfrak m$ is maximal too and we obtain a morphism $bar f:mathbb F_pto k$.

Since $k$ is finitely generated over $mathbb F_p$ and is a field, it is actually a finite extension ("Zariski's version of the Nullstellensatz") and thus $k$ is (set-theoretically!) finite.



Edit

Considering the comments below , I had better state explicitly the theorem I have used.
Theorem

Let $f:Ato B$ be a finitely generated $A$-algebra.

If $A$ is a Jacobson ring, , then $B$ is also Jacobson and for every maximal ideal $mathfrak msubset B$ the ideal $f^{-1}(mathfrak m)subset A$ is also maximal.



New Edit

Here is a proof in the style of algebraic geometry, due to Akaki Tikaradze.

If $k$ has characteristic $p$ we conclude as above, using Zariski.

If $operatorname{char}k=0$, the morphism $f:mathbb Zto k :$ is injective, so $A$ has no $mathbb Z$-torsion and is thus $mathbb Z$-flat.

But then $operatorname{Spec}(k)tooperatorname{Spec}(mathbb Z)$ is open (flat+finite presentation $implies$ open), i.e. $(0)inoperatorname{Spec}(mathbb Z)$ is open. Contradiction.

(This proof is absurdly sophisticated but it will probably appeal to scheme-theory addicts.
Take it as some kind of joke...)






share|cite|improve this answer











$endgroup$













  • $begingroup$
    Georges: My apologies....I thought this made perfect sense this morning but now I suddenly don't see it anymore. ${mathbb Q}$ is also a Jacobson ring, but the inverse image of (0) under the map ${mathbb Z}rightarrow {mathbb Q}$ is not a maximal ideal. What additional property are you using?
    $endgroup$
    – WillO
    May 24 '12 at 3:21










  • $begingroup$
    Dear Will, I'm using that $k$ is finitely generated over $mathbb Z$ (and that $mathbb Z $ is Jacobson).This is crucial as your counterexample correctly shows. An online reference is De Jong and collaborators' Stack Project, [Chapter 7: Commutative Algebra, Proposition 32.18 ](math.columbia.edu/algebraic_geometry/stacks-git/algebra.pdf)
    $endgroup$
    – Georges Elencwajg
    May 24 '12 at 5:58












  • $begingroup$
    Another reference is the generalization of the nullstellensatz stated here: en.wikipedia.org/wiki/Nullstellensatz .
    $endgroup$
    – Justin Young
    May 24 '12 at 12:12










  • $begingroup$
    Georges: Yes, as I said, I understood this the first time, but somehow when I reread it late at night I lost track of the main point. I remember why it made sense now! Thanks for being patient with me.
    $endgroup$
    – WillO
    May 24 '12 at 12:43










  • $begingroup$
    PS: You wrote "Since "k" is a Jacobson ring" but I think you meant "Since ${mathbb Z}$ is a Jacobson ring"; I think you need this to apply the Nullstellensatz.
    $endgroup$
    – WillO
    May 24 '12 at 14:28



















9












$begingroup$

In case you are interested, here is a proof which also uses Zariski's lemma but no difficult theorems about Jacobson rings.



Write $R$ for the image of the unique ring homomorphism $mathbb{Z} to k$, so that $k$ is a finitely generated $R$-algebra and hence a finite extension of the fraction field of $R$ by Zariski's lemma. Thus it suffices to show that $R = mathbb{F}_p$, which is to say $k$ has positive characteristic. If $R = mathbb{Z}$, meaning $k$ has characteristic zero, then $k$ is a number field which is a finitely generated ring. But this is impossible: if we write $k = mathbb{Z}[alpha_1,dots,alpha_r]$, then one can choose $n in mathbb{Z}$ so that all the denominators of coefficients in the minimal polynomials over $mathbb{Q}$ of $alpha_1,dots,alpha_r$ divide $n$. This implies that $k$ is integral over $mathbb{Z}[1/n]$. Then $mathbb{Z}[1/n]$ must be a field, which is absurd.






share|cite|improve this answer











$endgroup$





















    2












    $begingroup$

    When I read the proofs suggested in the answers half a year ago, I had the problem that I did not understand them because of a lack of algebra-knowledge. That's why I would like to share a (quite detailed) proof of the theorem which I finally understood:



    I will use Zariski's Lemma: Let $mathbb{K}$ be a field and $A$ be a finitely generated $mathbb{K}$-algebra. If $A$ is a field, then it is a finite degree extension of $mathbb{K}$.



    Now the proof: There exist elements $a_1,...,a_n$ such that $R cong mathbb{Z} [a_1,...,a_n]$. In order to simplify our notation we may assume that $R = mathbb{Z} [a_1,...,a_n]$. Why? - If $phi colon R to mathbb{Z} [a_1,...,a_n]$ is the isomorphism (that means $phi(frak{m}) subseteq mathbb{Z} [a_1,...,a_n]$ is a maximal ideal) and $pi colon mathbb{Z} [a_1,...,a_n] to mathbb{Z} [a_1,...,a_n]/(phi(frak{m}))$ denotes the canonical projection, then $pi circ phi colon R to mathbb{Z} [a_1,...,a_n]/(phi(frak{m}))$ is a surjective ring homomorphism with $ker(pi circ phi) = frak{m}$. Hence the fundamental homomorphism theorem tells us that $R/frak{m} cong mathbb{Z} [a_1,...,a_n]/(phi(frak{m}))$. So the ring of the left side is a finite field iff the ring on the right side is a finite field.



    Let $i$ be the only possible ring homomorphism $i colon mathbb{Z} to R/ frak{m}$ and let $S := i( mathbb{Z}) = {z + m mid z in mathbb{Z} }$. Note that "$S$" is a ring. Since $mathbb{Z}[a_1,...,a_n] = { f(a_1,...,a_n) mid f in mathbb{Z}[x_1,...,x_n] }$, it is easy to see that $R/frak{m} = mathbb{Z}[a_1,...,a_n] / frak{m} =$ $S[bar a_1, ..., bar a_n]$, where $bar a_i = a_i + frak{m}$, is a finitely generated $S$-algebra, hence a finite degree extension of the quotient field of $S$ (according to Zariski's lemma).
    Let's assume that $i$ is injective $Longleftrightarrow mathbb{Z} = S$.
    Thus $R/ frak{m}$ is a finite degree extension of $mathbb{Q}$. From this we conclude that the elements $bar a_1,..., bar a_n$ with $R/ mathfrak{m} = mathbb{Z}[bar a_1,..., bar a_n]$ are algebraic over $mathbb{Q}$ which means that they are zeros of certain monic polynomials with coefficients in $mathbb{Q}$. Now let $k$ be the product of the denominators of the coefficients of the minimal polynomials of the $bar a_i$ over $mathbb{Q}$. As a consequence, the coefficients of the minimal polynomials can be considered as elements of $mathbb{Z}[ frac{1}{k}]$, hence the $bar a_i$ are integral over $mathbb{Z}[ frac{1}{k}]$ and so the whole $R/ mathfrak{m}$ is integral over $mathbb{Z}[ frac{1}{k}]$.



    Let $a in mathbb{Z}[ frac{1}{k}], a neq 0$ arbitrary, thus $a in mathbb{Q} subseteq R/ frak{m}$. Since $R/ frak{m}$ is (a field and) integral over $mathbb{Z}[ frac{1}{k}]$, there exist $m in mathbb{N}, c_0,...,c_{m-1} in mathbb{Z}[ frac{1}{k}]$ such that
    $$a^{-m} + c_{m-1} a^{-(m-1)}+cdots+c_0 = 0,$$ and thus
    $$a^{-1}=-(c_{n-1}+cdots+c_0 a^{m-1}) in mathbb{Z}[ frac{1}{k}].$$
    Hence $mathbb{Z}[ frac{1}{k}]$ is a field, but that's a contradiction, since any prime number $q$, which does not divide $k$, cannot be invertible in $mathbb{Z}[ frac{1}{k}]$, because there is no element like $frac{1}{q}$. So $i$ cannot be injective.



    Thus there exists a $p in mathbb{N}$ prime such that $S = mathbb{Z}/ p mathbb{Z}$, hence $S$ is a finite field. From above we conclude that $R/ frak{m}$ is a finite degree extension of a finite field, thus a finite field. $Box$






    share|cite|improve this answer











    $endgroup$





















      2












      $begingroup$

      This is a mild generalization of Justin Campbell's answer, and the proof is almost the same as that of Proposition 7.8 in Atiyah and MacDonald's Introduction to Commutative Algebra. (This proposition is sometimes called the Artin-Tate Lemma.) The notation has been designed to make the analogy as clear as possible.




      Let $Asubset Bsubset C$ be a tower of domains such that $B$ is the field of fractions of $A$, and $C$ is a field which is finitely generated as an $A$-algebra. Then (a) $C$ is a finite extension of $B$, and (b) $B$ is a finitely generated $A$-algebra.




      As (a) results immediately from Zariski's Lemma, it suffices to prove (b). The following statement is ironically stronger and easier to prove than (b):




      Assume that $C=A[x_1,dots,x_m]$; that $y_1,dots,y_n$ is a $B$-basis of $C$ with $y_1=1$; and that we have $$x_i=sum_jb_{ij}y_j,quad y_iy_j=sum_kb_{ijk}y_k$$ with $b_{ij}$, $b_{ijk}$ in $B$. Then the $A$-algebra $B$ is generated by the $b_{ij}$ and the $b_{ijk}$.




      The proof is straightforward, and left to the reader.






      share|cite|improve this answer









      $endgroup$













        Your Answer





        StackExchange.ifUsing("editor", function () {
        return StackExchange.using("mathjaxEditing", function () {
        StackExchange.MarkdownEditor.creationCallbacks.add(function (editor, postfix) {
        StackExchange.mathjaxEditing.prepareWmdForMathJax(editor, postfix, [["$", "$"], ["\\(","\\)"]]);
        });
        });
        }, "mathjax-editing");

        StackExchange.ready(function() {
        var channelOptions = {
        tags: "".split(" "),
        id: "69"
        };
        initTagRenderer("".split(" "), "".split(" "), channelOptions);

        StackExchange.using("externalEditor", function() {
        // Have to fire editor after snippets, if snippets enabled
        if (StackExchange.settings.snippets.snippetsEnabled) {
        StackExchange.using("snippets", function() {
        createEditor();
        });
        }
        else {
        createEditor();
        }
        });

        function createEditor() {
        StackExchange.prepareEditor({
        heartbeatType: 'answer',
        autoActivateHeartbeat: false,
        convertImagesToLinks: true,
        noModals: true,
        showLowRepImageUploadWarning: true,
        reputationToPostImages: 10,
        bindNavPrevention: true,
        postfix: "",
        imageUploader: {
        brandingHtml: "Powered by u003ca class="icon-imgur-white" href="https://imgur.com/"u003eu003c/au003e",
        contentPolicyHtml: "User contributions licensed under u003ca href="https://creativecommons.org/licenses/by-sa/3.0/"u003ecc by-sa 3.0 with attribution requiredu003c/au003e u003ca href="https://stackoverflow.com/legal/content-policy"u003e(content policy)u003c/au003e",
        allowUrls: true
        },
        noCode: true, onDemand: true,
        discardSelector: ".discard-answer"
        ,immediatelyShowMarkdownHelp:true
        });


        }
        });














        draft saved

        draft discarded


















        StackExchange.ready(
        function () {
        StackExchange.openid.initPostLogin('.new-post-login', 'https%3a%2f%2fmath.stackexchange.com%2fquestions%2f148745%2ffields-finitely-generated-as-mathbb-z-algebras-are-finite%23new-answer', 'question_page');
        }
        );

        Post as a guest















        Required, but never shown

























        4 Answers
        4






        active

        oldest

        votes








        4 Answers
        4






        active

        oldest

        votes









        active

        oldest

        votes






        active

        oldest

        votes









        18












        $begingroup$

        Yes!

        Consider the morphism
        $f:mathbb Zto k$ and the ideal $mathfrak m=f^{-1}(0)subset mathbb Z$.

        Since $mathbb Z$ is a Jacobson ring and $(0)subset k$ is maximal, $mathfrak m$ is maximal too and we obtain a morphism $bar f:mathbb F_pto k$.

        Since $k$ is finitely generated over $mathbb F_p$ and is a field, it is actually a finite extension ("Zariski's version of the Nullstellensatz") and thus $k$ is (set-theoretically!) finite.



        Edit

        Considering the comments below , I had better state explicitly the theorem I have used.
        Theorem

        Let $f:Ato B$ be a finitely generated $A$-algebra.

        If $A$ is a Jacobson ring, , then $B$ is also Jacobson and for every maximal ideal $mathfrak msubset B$ the ideal $f^{-1}(mathfrak m)subset A$ is also maximal.



        New Edit

        Here is a proof in the style of algebraic geometry, due to Akaki Tikaradze.

        If $k$ has characteristic $p$ we conclude as above, using Zariski.

        If $operatorname{char}k=0$, the morphism $f:mathbb Zto k :$ is injective, so $A$ has no $mathbb Z$-torsion and is thus $mathbb Z$-flat.

        But then $operatorname{Spec}(k)tooperatorname{Spec}(mathbb Z)$ is open (flat+finite presentation $implies$ open), i.e. $(0)inoperatorname{Spec}(mathbb Z)$ is open. Contradiction.

        (This proof is absurdly sophisticated but it will probably appeal to scheme-theory addicts.
        Take it as some kind of joke...)






        share|cite|improve this answer











        $endgroup$













        • $begingroup$
          Georges: My apologies....I thought this made perfect sense this morning but now I suddenly don't see it anymore. ${mathbb Q}$ is also a Jacobson ring, but the inverse image of (0) under the map ${mathbb Z}rightarrow {mathbb Q}$ is not a maximal ideal. What additional property are you using?
          $endgroup$
          – WillO
          May 24 '12 at 3:21










        • $begingroup$
          Dear Will, I'm using that $k$ is finitely generated over $mathbb Z$ (and that $mathbb Z $ is Jacobson).This is crucial as your counterexample correctly shows. An online reference is De Jong and collaborators' Stack Project, [Chapter 7: Commutative Algebra, Proposition 32.18 ](math.columbia.edu/algebraic_geometry/stacks-git/algebra.pdf)
          $endgroup$
          – Georges Elencwajg
          May 24 '12 at 5:58












        • $begingroup$
          Another reference is the generalization of the nullstellensatz stated here: en.wikipedia.org/wiki/Nullstellensatz .
          $endgroup$
          – Justin Young
          May 24 '12 at 12:12










        • $begingroup$
          Georges: Yes, as I said, I understood this the first time, but somehow when I reread it late at night I lost track of the main point. I remember why it made sense now! Thanks for being patient with me.
          $endgroup$
          – WillO
          May 24 '12 at 12:43










        • $begingroup$
          PS: You wrote "Since "k" is a Jacobson ring" but I think you meant "Since ${mathbb Z}$ is a Jacobson ring"; I think you need this to apply the Nullstellensatz.
          $endgroup$
          – WillO
          May 24 '12 at 14:28
















        18












        $begingroup$

        Yes!

        Consider the morphism
        $f:mathbb Zto k$ and the ideal $mathfrak m=f^{-1}(0)subset mathbb Z$.

        Since $mathbb Z$ is a Jacobson ring and $(0)subset k$ is maximal, $mathfrak m$ is maximal too and we obtain a morphism $bar f:mathbb F_pto k$.

        Since $k$ is finitely generated over $mathbb F_p$ and is a field, it is actually a finite extension ("Zariski's version of the Nullstellensatz") and thus $k$ is (set-theoretically!) finite.



        Edit

        Considering the comments below , I had better state explicitly the theorem I have used.
        Theorem

        Let $f:Ato B$ be a finitely generated $A$-algebra.

        If $A$ is a Jacobson ring, , then $B$ is also Jacobson and for every maximal ideal $mathfrak msubset B$ the ideal $f^{-1}(mathfrak m)subset A$ is also maximal.



        New Edit

        Here is a proof in the style of algebraic geometry, due to Akaki Tikaradze.

        If $k$ has characteristic $p$ we conclude as above, using Zariski.

        If $operatorname{char}k=0$, the morphism $f:mathbb Zto k :$ is injective, so $A$ has no $mathbb Z$-torsion and is thus $mathbb Z$-flat.

        But then $operatorname{Spec}(k)tooperatorname{Spec}(mathbb Z)$ is open (flat+finite presentation $implies$ open), i.e. $(0)inoperatorname{Spec}(mathbb Z)$ is open. Contradiction.

        (This proof is absurdly sophisticated but it will probably appeal to scheme-theory addicts.
        Take it as some kind of joke...)






        share|cite|improve this answer











        $endgroup$













        • $begingroup$
          Georges: My apologies....I thought this made perfect sense this morning but now I suddenly don't see it anymore. ${mathbb Q}$ is also a Jacobson ring, but the inverse image of (0) under the map ${mathbb Z}rightarrow {mathbb Q}$ is not a maximal ideal. What additional property are you using?
          $endgroup$
          – WillO
          May 24 '12 at 3:21










        • $begingroup$
          Dear Will, I'm using that $k$ is finitely generated over $mathbb Z$ (and that $mathbb Z $ is Jacobson).This is crucial as your counterexample correctly shows. An online reference is De Jong and collaborators' Stack Project, [Chapter 7: Commutative Algebra, Proposition 32.18 ](math.columbia.edu/algebraic_geometry/stacks-git/algebra.pdf)
          $endgroup$
          – Georges Elencwajg
          May 24 '12 at 5:58












        • $begingroup$
          Another reference is the generalization of the nullstellensatz stated here: en.wikipedia.org/wiki/Nullstellensatz .
          $endgroup$
          – Justin Young
          May 24 '12 at 12:12










        • $begingroup$
          Georges: Yes, as I said, I understood this the first time, but somehow when I reread it late at night I lost track of the main point. I remember why it made sense now! Thanks for being patient with me.
          $endgroup$
          – WillO
          May 24 '12 at 12:43










        • $begingroup$
          PS: You wrote "Since "k" is a Jacobson ring" but I think you meant "Since ${mathbb Z}$ is a Jacobson ring"; I think you need this to apply the Nullstellensatz.
          $endgroup$
          – WillO
          May 24 '12 at 14:28














        18












        18








        18





        $begingroup$

        Yes!

        Consider the morphism
        $f:mathbb Zto k$ and the ideal $mathfrak m=f^{-1}(0)subset mathbb Z$.

        Since $mathbb Z$ is a Jacobson ring and $(0)subset k$ is maximal, $mathfrak m$ is maximal too and we obtain a morphism $bar f:mathbb F_pto k$.

        Since $k$ is finitely generated over $mathbb F_p$ and is a field, it is actually a finite extension ("Zariski's version of the Nullstellensatz") and thus $k$ is (set-theoretically!) finite.



        Edit

        Considering the comments below , I had better state explicitly the theorem I have used.
        Theorem

        Let $f:Ato B$ be a finitely generated $A$-algebra.

        If $A$ is a Jacobson ring, , then $B$ is also Jacobson and for every maximal ideal $mathfrak msubset B$ the ideal $f^{-1}(mathfrak m)subset A$ is also maximal.



        New Edit

        Here is a proof in the style of algebraic geometry, due to Akaki Tikaradze.

        If $k$ has characteristic $p$ we conclude as above, using Zariski.

        If $operatorname{char}k=0$, the morphism $f:mathbb Zto k :$ is injective, so $A$ has no $mathbb Z$-torsion and is thus $mathbb Z$-flat.

        But then $operatorname{Spec}(k)tooperatorname{Spec}(mathbb Z)$ is open (flat+finite presentation $implies$ open), i.e. $(0)inoperatorname{Spec}(mathbb Z)$ is open. Contradiction.

        (This proof is absurdly sophisticated but it will probably appeal to scheme-theory addicts.
        Take it as some kind of joke...)






        share|cite|improve this answer











        $endgroup$



        Yes!

        Consider the morphism
        $f:mathbb Zto k$ and the ideal $mathfrak m=f^{-1}(0)subset mathbb Z$.

        Since $mathbb Z$ is a Jacobson ring and $(0)subset k$ is maximal, $mathfrak m$ is maximal too and we obtain a morphism $bar f:mathbb F_pto k$.

        Since $k$ is finitely generated over $mathbb F_p$ and is a field, it is actually a finite extension ("Zariski's version of the Nullstellensatz") and thus $k$ is (set-theoretically!) finite.



        Edit

        Considering the comments below , I had better state explicitly the theorem I have used.
        Theorem

        Let $f:Ato B$ be a finitely generated $A$-algebra.

        If $A$ is a Jacobson ring, , then $B$ is also Jacobson and for every maximal ideal $mathfrak msubset B$ the ideal $f^{-1}(mathfrak m)subset A$ is also maximal.



        New Edit

        Here is a proof in the style of algebraic geometry, due to Akaki Tikaradze.

        If $k$ has characteristic $p$ we conclude as above, using Zariski.

        If $operatorname{char}k=0$, the morphism $f:mathbb Zto k :$ is injective, so $A$ has no $mathbb Z$-torsion and is thus $mathbb Z$-flat.

        But then $operatorname{Spec}(k)tooperatorname{Spec}(mathbb Z)$ is open (flat+finite presentation $implies$ open), i.e. $(0)inoperatorname{Spec}(mathbb Z)$ is open. Contradiction.

        (This proof is absurdly sophisticated but it will probably appeal to scheme-theory addicts.
        Take it as some kind of joke...)







        share|cite|improve this answer














        share|cite|improve this answer



        share|cite|improve this answer








        edited Dec 5 '18 at 15:02









        user26857

        39.3k124183




        39.3k124183










        answered May 23 '12 at 14:41









        Georges ElencwajgGeorges Elencwajg

        119k7181332




        119k7181332












        • $begingroup$
          Georges: My apologies....I thought this made perfect sense this morning but now I suddenly don't see it anymore. ${mathbb Q}$ is also a Jacobson ring, but the inverse image of (0) under the map ${mathbb Z}rightarrow {mathbb Q}$ is not a maximal ideal. What additional property are you using?
          $endgroup$
          – WillO
          May 24 '12 at 3:21










        • $begingroup$
          Dear Will, I'm using that $k$ is finitely generated over $mathbb Z$ (and that $mathbb Z $ is Jacobson).This is crucial as your counterexample correctly shows. An online reference is De Jong and collaborators' Stack Project, [Chapter 7: Commutative Algebra, Proposition 32.18 ](math.columbia.edu/algebraic_geometry/stacks-git/algebra.pdf)
          $endgroup$
          – Georges Elencwajg
          May 24 '12 at 5:58












        • $begingroup$
          Another reference is the generalization of the nullstellensatz stated here: en.wikipedia.org/wiki/Nullstellensatz .
          $endgroup$
          – Justin Young
          May 24 '12 at 12:12










        • $begingroup$
          Georges: Yes, as I said, I understood this the first time, but somehow when I reread it late at night I lost track of the main point. I remember why it made sense now! Thanks for being patient with me.
          $endgroup$
          – WillO
          May 24 '12 at 12:43










        • $begingroup$
          PS: You wrote "Since "k" is a Jacobson ring" but I think you meant "Since ${mathbb Z}$ is a Jacobson ring"; I think you need this to apply the Nullstellensatz.
          $endgroup$
          – WillO
          May 24 '12 at 14:28


















        • $begingroup$
          Georges: My apologies....I thought this made perfect sense this morning but now I suddenly don't see it anymore. ${mathbb Q}$ is also a Jacobson ring, but the inverse image of (0) under the map ${mathbb Z}rightarrow {mathbb Q}$ is not a maximal ideal. What additional property are you using?
          $endgroup$
          – WillO
          May 24 '12 at 3:21










        • $begingroup$
          Dear Will, I'm using that $k$ is finitely generated over $mathbb Z$ (and that $mathbb Z $ is Jacobson).This is crucial as your counterexample correctly shows. An online reference is De Jong and collaborators' Stack Project, [Chapter 7: Commutative Algebra, Proposition 32.18 ](math.columbia.edu/algebraic_geometry/stacks-git/algebra.pdf)
          $endgroup$
          – Georges Elencwajg
          May 24 '12 at 5:58












        • $begingroup$
          Another reference is the generalization of the nullstellensatz stated here: en.wikipedia.org/wiki/Nullstellensatz .
          $endgroup$
          – Justin Young
          May 24 '12 at 12:12










        • $begingroup$
          Georges: Yes, as I said, I understood this the first time, but somehow when I reread it late at night I lost track of the main point. I remember why it made sense now! Thanks for being patient with me.
          $endgroup$
          – WillO
          May 24 '12 at 12:43










        • $begingroup$
          PS: You wrote "Since "k" is a Jacobson ring" but I think you meant "Since ${mathbb Z}$ is a Jacobson ring"; I think you need this to apply the Nullstellensatz.
          $endgroup$
          – WillO
          May 24 '12 at 14:28
















        $begingroup$
        Georges: My apologies....I thought this made perfect sense this morning but now I suddenly don't see it anymore. ${mathbb Q}$ is also a Jacobson ring, but the inverse image of (0) under the map ${mathbb Z}rightarrow {mathbb Q}$ is not a maximal ideal. What additional property are you using?
        $endgroup$
        – WillO
        May 24 '12 at 3:21




        $begingroup$
        Georges: My apologies....I thought this made perfect sense this morning but now I suddenly don't see it anymore. ${mathbb Q}$ is also a Jacobson ring, but the inverse image of (0) under the map ${mathbb Z}rightarrow {mathbb Q}$ is not a maximal ideal. What additional property are you using?
        $endgroup$
        – WillO
        May 24 '12 at 3:21












        $begingroup$
        Dear Will, I'm using that $k$ is finitely generated over $mathbb Z$ (and that $mathbb Z $ is Jacobson).This is crucial as your counterexample correctly shows. An online reference is De Jong and collaborators' Stack Project, [Chapter 7: Commutative Algebra, Proposition 32.18 ](math.columbia.edu/algebraic_geometry/stacks-git/algebra.pdf)
        $endgroup$
        – Georges Elencwajg
        May 24 '12 at 5:58






        $begingroup$
        Dear Will, I'm using that $k$ is finitely generated over $mathbb Z$ (and that $mathbb Z $ is Jacobson).This is crucial as your counterexample correctly shows. An online reference is De Jong and collaborators' Stack Project, [Chapter 7: Commutative Algebra, Proposition 32.18 ](math.columbia.edu/algebraic_geometry/stacks-git/algebra.pdf)
        $endgroup$
        – Georges Elencwajg
        May 24 '12 at 5:58














        $begingroup$
        Another reference is the generalization of the nullstellensatz stated here: en.wikipedia.org/wiki/Nullstellensatz .
        $endgroup$
        – Justin Young
        May 24 '12 at 12:12




        $begingroup$
        Another reference is the generalization of the nullstellensatz stated here: en.wikipedia.org/wiki/Nullstellensatz .
        $endgroup$
        – Justin Young
        May 24 '12 at 12:12












        $begingroup$
        Georges: Yes, as I said, I understood this the first time, but somehow when I reread it late at night I lost track of the main point. I remember why it made sense now! Thanks for being patient with me.
        $endgroup$
        – WillO
        May 24 '12 at 12:43




        $begingroup$
        Georges: Yes, as I said, I understood this the first time, but somehow when I reread it late at night I lost track of the main point. I remember why it made sense now! Thanks for being patient with me.
        $endgroup$
        – WillO
        May 24 '12 at 12:43












        $begingroup$
        PS: You wrote "Since "k" is a Jacobson ring" but I think you meant "Since ${mathbb Z}$ is a Jacobson ring"; I think you need this to apply the Nullstellensatz.
        $endgroup$
        – WillO
        May 24 '12 at 14:28




        $begingroup$
        PS: You wrote "Since "k" is a Jacobson ring" but I think you meant "Since ${mathbb Z}$ is a Jacobson ring"; I think you need this to apply the Nullstellensatz.
        $endgroup$
        – WillO
        May 24 '12 at 14:28











        9












        $begingroup$

        In case you are interested, here is a proof which also uses Zariski's lemma but no difficult theorems about Jacobson rings.



        Write $R$ for the image of the unique ring homomorphism $mathbb{Z} to k$, so that $k$ is a finitely generated $R$-algebra and hence a finite extension of the fraction field of $R$ by Zariski's lemma. Thus it suffices to show that $R = mathbb{F}_p$, which is to say $k$ has positive characteristic. If $R = mathbb{Z}$, meaning $k$ has characteristic zero, then $k$ is a number field which is a finitely generated ring. But this is impossible: if we write $k = mathbb{Z}[alpha_1,dots,alpha_r]$, then one can choose $n in mathbb{Z}$ so that all the denominators of coefficients in the minimal polynomials over $mathbb{Q}$ of $alpha_1,dots,alpha_r$ divide $n$. This implies that $k$ is integral over $mathbb{Z}[1/n]$. Then $mathbb{Z}[1/n]$ must be a field, which is absurd.






        share|cite|improve this answer











        $endgroup$


















          9












          $begingroup$

          In case you are interested, here is a proof which also uses Zariski's lemma but no difficult theorems about Jacobson rings.



          Write $R$ for the image of the unique ring homomorphism $mathbb{Z} to k$, so that $k$ is a finitely generated $R$-algebra and hence a finite extension of the fraction field of $R$ by Zariski's lemma. Thus it suffices to show that $R = mathbb{F}_p$, which is to say $k$ has positive characteristic. If $R = mathbb{Z}$, meaning $k$ has characteristic zero, then $k$ is a number field which is a finitely generated ring. But this is impossible: if we write $k = mathbb{Z}[alpha_1,dots,alpha_r]$, then one can choose $n in mathbb{Z}$ so that all the denominators of coefficients in the minimal polynomials over $mathbb{Q}$ of $alpha_1,dots,alpha_r$ divide $n$. This implies that $k$ is integral over $mathbb{Z}[1/n]$. Then $mathbb{Z}[1/n]$ must be a field, which is absurd.






          share|cite|improve this answer











          $endgroup$
















            9












            9








            9





            $begingroup$

            In case you are interested, here is a proof which also uses Zariski's lemma but no difficult theorems about Jacobson rings.



            Write $R$ for the image of the unique ring homomorphism $mathbb{Z} to k$, so that $k$ is a finitely generated $R$-algebra and hence a finite extension of the fraction field of $R$ by Zariski's lemma. Thus it suffices to show that $R = mathbb{F}_p$, which is to say $k$ has positive characteristic. If $R = mathbb{Z}$, meaning $k$ has characteristic zero, then $k$ is a number field which is a finitely generated ring. But this is impossible: if we write $k = mathbb{Z}[alpha_1,dots,alpha_r]$, then one can choose $n in mathbb{Z}$ so that all the denominators of coefficients in the minimal polynomials over $mathbb{Q}$ of $alpha_1,dots,alpha_r$ divide $n$. This implies that $k$ is integral over $mathbb{Z}[1/n]$. Then $mathbb{Z}[1/n]$ must be a field, which is absurd.






            share|cite|improve this answer











            $endgroup$



            In case you are interested, here is a proof which also uses Zariski's lemma but no difficult theorems about Jacobson rings.



            Write $R$ for the image of the unique ring homomorphism $mathbb{Z} to k$, so that $k$ is a finitely generated $R$-algebra and hence a finite extension of the fraction field of $R$ by Zariski's lemma. Thus it suffices to show that $R = mathbb{F}_p$, which is to say $k$ has positive characteristic. If $R = mathbb{Z}$, meaning $k$ has characteristic zero, then $k$ is a number field which is a finitely generated ring. But this is impossible: if we write $k = mathbb{Z}[alpha_1,dots,alpha_r]$, then one can choose $n in mathbb{Z}$ so that all the denominators of coefficients in the minimal polynomials over $mathbb{Q}$ of $alpha_1,dots,alpha_r$ divide $n$. This implies that $k$ is integral over $mathbb{Z}[1/n]$. Then $mathbb{Z}[1/n]$ must be a field, which is absurd.







            share|cite|improve this answer














            share|cite|improve this answer



            share|cite|improve this answer








            edited Nov 13 '14 at 0:27









            user26857

            39.3k124183




            39.3k124183










            answered May 24 '12 at 15:59









            Justin CampbellJustin Campbell

            3,9841540




            3,9841540























                2












                $begingroup$

                When I read the proofs suggested in the answers half a year ago, I had the problem that I did not understand them because of a lack of algebra-knowledge. That's why I would like to share a (quite detailed) proof of the theorem which I finally understood:



                I will use Zariski's Lemma: Let $mathbb{K}$ be a field and $A$ be a finitely generated $mathbb{K}$-algebra. If $A$ is a field, then it is a finite degree extension of $mathbb{K}$.



                Now the proof: There exist elements $a_1,...,a_n$ such that $R cong mathbb{Z} [a_1,...,a_n]$. In order to simplify our notation we may assume that $R = mathbb{Z} [a_1,...,a_n]$. Why? - If $phi colon R to mathbb{Z} [a_1,...,a_n]$ is the isomorphism (that means $phi(frak{m}) subseteq mathbb{Z} [a_1,...,a_n]$ is a maximal ideal) and $pi colon mathbb{Z} [a_1,...,a_n] to mathbb{Z} [a_1,...,a_n]/(phi(frak{m}))$ denotes the canonical projection, then $pi circ phi colon R to mathbb{Z} [a_1,...,a_n]/(phi(frak{m}))$ is a surjective ring homomorphism with $ker(pi circ phi) = frak{m}$. Hence the fundamental homomorphism theorem tells us that $R/frak{m} cong mathbb{Z} [a_1,...,a_n]/(phi(frak{m}))$. So the ring of the left side is a finite field iff the ring on the right side is a finite field.



                Let $i$ be the only possible ring homomorphism $i colon mathbb{Z} to R/ frak{m}$ and let $S := i( mathbb{Z}) = {z + m mid z in mathbb{Z} }$. Note that "$S$" is a ring. Since $mathbb{Z}[a_1,...,a_n] = { f(a_1,...,a_n) mid f in mathbb{Z}[x_1,...,x_n] }$, it is easy to see that $R/frak{m} = mathbb{Z}[a_1,...,a_n] / frak{m} =$ $S[bar a_1, ..., bar a_n]$, where $bar a_i = a_i + frak{m}$, is a finitely generated $S$-algebra, hence a finite degree extension of the quotient field of $S$ (according to Zariski's lemma).
                Let's assume that $i$ is injective $Longleftrightarrow mathbb{Z} = S$.
                Thus $R/ frak{m}$ is a finite degree extension of $mathbb{Q}$. From this we conclude that the elements $bar a_1,..., bar a_n$ with $R/ mathfrak{m} = mathbb{Z}[bar a_1,..., bar a_n]$ are algebraic over $mathbb{Q}$ which means that they are zeros of certain monic polynomials with coefficients in $mathbb{Q}$. Now let $k$ be the product of the denominators of the coefficients of the minimal polynomials of the $bar a_i$ over $mathbb{Q}$. As a consequence, the coefficients of the minimal polynomials can be considered as elements of $mathbb{Z}[ frac{1}{k}]$, hence the $bar a_i$ are integral over $mathbb{Z}[ frac{1}{k}]$ and so the whole $R/ mathfrak{m}$ is integral over $mathbb{Z}[ frac{1}{k}]$.



                Let $a in mathbb{Z}[ frac{1}{k}], a neq 0$ arbitrary, thus $a in mathbb{Q} subseteq R/ frak{m}$. Since $R/ frak{m}$ is (a field and) integral over $mathbb{Z}[ frac{1}{k}]$, there exist $m in mathbb{N}, c_0,...,c_{m-1} in mathbb{Z}[ frac{1}{k}]$ such that
                $$a^{-m} + c_{m-1} a^{-(m-1)}+cdots+c_0 = 0,$$ and thus
                $$a^{-1}=-(c_{n-1}+cdots+c_0 a^{m-1}) in mathbb{Z}[ frac{1}{k}].$$
                Hence $mathbb{Z}[ frac{1}{k}]$ is a field, but that's a contradiction, since any prime number $q$, which does not divide $k$, cannot be invertible in $mathbb{Z}[ frac{1}{k}]$, because there is no element like $frac{1}{q}$. So $i$ cannot be injective.



                Thus there exists a $p in mathbb{N}$ prime such that $S = mathbb{Z}/ p mathbb{Z}$, hence $S$ is a finite field. From above we conclude that $R/ frak{m}$ is a finite degree extension of a finite field, thus a finite field. $Box$






                share|cite|improve this answer











                $endgroup$


















                  2












                  $begingroup$

                  When I read the proofs suggested in the answers half a year ago, I had the problem that I did not understand them because of a lack of algebra-knowledge. That's why I would like to share a (quite detailed) proof of the theorem which I finally understood:



                  I will use Zariski's Lemma: Let $mathbb{K}$ be a field and $A$ be a finitely generated $mathbb{K}$-algebra. If $A$ is a field, then it is a finite degree extension of $mathbb{K}$.



                  Now the proof: There exist elements $a_1,...,a_n$ such that $R cong mathbb{Z} [a_1,...,a_n]$. In order to simplify our notation we may assume that $R = mathbb{Z} [a_1,...,a_n]$. Why? - If $phi colon R to mathbb{Z} [a_1,...,a_n]$ is the isomorphism (that means $phi(frak{m}) subseteq mathbb{Z} [a_1,...,a_n]$ is a maximal ideal) and $pi colon mathbb{Z} [a_1,...,a_n] to mathbb{Z} [a_1,...,a_n]/(phi(frak{m}))$ denotes the canonical projection, then $pi circ phi colon R to mathbb{Z} [a_1,...,a_n]/(phi(frak{m}))$ is a surjective ring homomorphism with $ker(pi circ phi) = frak{m}$. Hence the fundamental homomorphism theorem tells us that $R/frak{m} cong mathbb{Z} [a_1,...,a_n]/(phi(frak{m}))$. So the ring of the left side is a finite field iff the ring on the right side is a finite field.



                  Let $i$ be the only possible ring homomorphism $i colon mathbb{Z} to R/ frak{m}$ and let $S := i( mathbb{Z}) = {z + m mid z in mathbb{Z} }$. Note that "$S$" is a ring. Since $mathbb{Z}[a_1,...,a_n] = { f(a_1,...,a_n) mid f in mathbb{Z}[x_1,...,x_n] }$, it is easy to see that $R/frak{m} = mathbb{Z}[a_1,...,a_n] / frak{m} =$ $S[bar a_1, ..., bar a_n]$, where $bar a_i = a_i + frak{m}$, is a finitely generated $S$-algebra, hence a finite degree extension of the quotient field of $S$ (according to Zariski's lemma).
                  Let's assume that $i$ is injective $Longleftrightarrow mathbb{Z} = S$.
                  Thus $R/ frak{m}$ is a finite degree extension of $mathbb{Q}$. From this we conclude that the elements $bar a_1,..., bar a_n$ with $R/ mathfrak{m} = mathbb{Z}[bar a_1,..., bar a_n]$ are algebraic over $mathbb{Q}$ which means that they are zeros of certain monic polynomials with coefficients in $mathbb{Q}$. Now let $k$ be the product of the denominators of the coefficients of the minimal polynomials of the $bar a_i$ over $mathbb{Q}$. As a consequence, the coefficients of the minimal polynomials can be considered as elements of $mathbb{Z}[ frac{1}{k}]$, hence the $bar a_i$ are integral over $mathbb{Z}[ frac{1}{k}]$ and so the whole $R/ mathfrak{m}$ is integral over $mathbb{Z}[ frac{1}{k}]$.



                  Let $a in mathbb{Z}[ frac{1}{k}], a neq 0$ arbitrary, thus $a in mathbb{Q} subseteq R/ frak{m}$. Since $R/ frak{m}$ is (a field and) integral over $mathbb{Z}[ frac{1}{k}]$, there exist $m in mathbb{N}, c_0,...,c_{m-1} in mathbb{Z}[ frac{1}{k}]$ such that
                  $$a^{-m} + c_{m-1} a^{-(m-1)}+cdots+c_0 = 0,$$ and thus
                  $$a^{-1}=-(c_{n-1}+cdots+c_0 a^{m-1}) in mathbb{Z}[ frac{1}{k}].$$
                  Hence $mathbb{Z}[ frac{1}{k}]$ is a field, but that's a contradiction, since any prime number $q$, which does not divide $k$, cannot be invertible in $mathbb{Z}[ frac{1}{k}]$, because there is no element like $frac{1}{q}$. So $i$ cannot be injective.



                  Thus there exists a $p in mathbb{N}$ prime such that $S = mathbb{Z}/ p mathbb{Z}$, hence $S$ is a finite field. From above we conclude that $R/ frak{m}$ is a finite degree extension of a finite field, thus a finite field. $Box$






                  share|cite|improve this answer











                  $endgroup$
















                    2












                    2








                    2





                    $begingroup$

                    When I read the proofs suggested in the answers half a year ago, I had the problem that I did not understand them because of a lack of algebra-knowledge. That's why I would like to share a (quite detailed) proof of the theorem which I finally understood:



                    I will use Zariski's Lemma: Let $mathbb{K}$ be a field and $A$ be a finitely generated $mathbb{K}$-algebra. If $A$ is a field, then it is a finite degree extension of $mathbb{K}$.



                    Now the proof: There exist elements $a_1,...,a_n$ such that $R cong mathbb{Z} [a_1,...,a_n]$. In order to simplify our notation we may assume that $R = mathbb{Z} [a_1,...,a_n]$. Why? - If $phi colon R to mathbb{Z} [a_1,...,a_n]$ is the isomorphism (that means $phi(frak{m}) subseteq mathbb{Z} [a_1,...,a_n]$ is a maximal ideal) and $pi colon mathbb{Z} [a_1,...,a_n] to mathbb{Z} [a_1,...,a_n]/(phi(frak{m}))$ denotes the canonical projection, then $pi circ phi colon R to mathbb{Z} [a_1,...,a_n]/(phi(frak{m}))$ is a surjective ring homomorphism with $ker(pi circ phi) = frak{m}$. Hence the fundamental homomorphism theorem tells us that $R/frak{m} cong mathbb{Z} [a_1,...,a_n]/(phi(frak{m}))$. So the ring of the left side is a finite field iff the ring on the right side is a finite field.



                    Let $i$ be the only possible ring homomorphism $i colon mathbb{Z} to R/ frak{m}$ and let $S := i( mathbb{Z}) = {z + m mid z in mathbb{Z} }$. Note that "$S$" is a ring. Since $mathbb{Z}[a_1,...,a_n] = { f(a_1,...,a_n) mid f in mathbb{Z}[x_1,...,x_n] }$, it is easy to see that $R/frak{m} = mathbb{Z}[a_1,...,a_n] / frak{m} =$ $S[bar a_1, ..., bar a_n]$, where $bar a_i = a_i + frak{m}$, is a finitely generated $S$-algebra, hence a finite degree extension of the quotient field of $S$ (according to Zariski's lemma).
                    Let's assume that $i$ is injective $Longleftrightarrow mathbb{Z} = S$.
                    Thus $R/ frak{m}$ is a finite degree extension of $mathbb{Q}$. From this we conclude that the elements $bar a_1,..., bar a_n$ with $R/ mathfrak{m} = mathbb{Z}[bar a_1,..., bar a_n]$ are algebraic over $mathbb{Q}$ which means that they are zeros of certain monic polynomials with coefficients in $mathbb{Q}$. Now let $k$ be the product of the denominators of the coefficients of the minimal polynomials of the $bar a_i$ over $mathbb{Q}$. As a consequence, the coefficients of the minimal polynomials can be considered as elements of $mathbb{Z}[ frac{1}{k}]$, hence the $bar a_i$ are integral over $mathbb{Z}[ frac{1}{k}]$ and so the whole $R/ mathfrak{m}$ is integral over $mathbb{Z}[ frac{1}{k}]$.



                    Let $a in mathbb{Z}[ frac{1}{k}], a neq 0$ arbitrary, thus $a in mathbb{Q} subseteq R/ frak{m}$. Since $R/ frak{m}$ is (a field and) integral over $mathbb{Z}[ frac{1}{k}]$, there exist $m in mathbb{N}, c_0,...,c_{m-1} in mathbb{Z}[ frac{1}{k}]$ such that
                    $$a^{-m} + c_{m-1} a^{-(m-1)}+cdots+c_0 = 0,$$ and thus
                    $$a^{-1}=-(c_{n-1}+cdots+c_0 a^{m-1}) in mathbb{Z}[ frac{1}{k}].$$
                    Hence $mathbb{Z}[ frac{1}{k}]$ is a field, but that's a contradiction, since any prime number $q$, which does not divide $k$, cannot be invertible in $mathbb{Z}[ frac{1}{k}]$, because there is no element like $frac{1}{q}$. So $i$ cannot be injective.



                    Thus there exists a $p in mathbb{N}$ prime such that $S = mathbb{Z}/ p mathbb{Z}$, hence $S$ is a finite field. From above we conclude that $R/ frak{m}$ is a finite degree extension of a finite field, thus a finite field. $Box$






                    share|cite|improve this answer











                    $endgroup$



                    When I read the proofs suggested in the answers half a year ago, I had the problem that I did not understand them because of a lack of algebra-knowledge. That's why I would like to share a (quite detailed) proof of the theorem which I finally understood:



                    I will use Zariski's Lemma: Let $mathbb{K}$ be a field and $A$ be a finitely generated $mathbb{K}$-algebra. If $A$ is a field, then it is a finite degree extension of $mathbb{K}$.



                    Now the proof: There exist elements $a_1,...,a_n$ such that $R cong mathbb{Z} [a_1,...,a_n]$. In order to simplify our notation we may assume that $R = mathbb{Z} [a_1,...,a_n]$. Why? - If $phi colon R to mathbb{Z} [a_1,...,a_n]$ is the isomorphism (that means $phi(frak{m}) subseteq mathbb{Z} [a_1,...,a_n]$ is a maximal ideal) and $pi colon mathbb{Z} [a_1,...,a_n] to mathbb{Z} [a_1,...,a_n]/(phi(frak{m}))$ denotes the canonical projection, then $pi circ phi colon R to mathbb{Z} [a_1,...,a_n]/(phi(frak{m}))$ is a surjective ring homomorphism with $ker(pi circ phi) = frak{m}$. Hence the fundamental homomorphism theorem tells us that $R/frak{m} cong mathbb{Z} [a_1,...,a_n]/(phi(frak{m}))$. So the ring of the left side is a finite field iff the ring on the right side is a finite field.



                    Let $i$ be the only possible ring homomorphism $i colon mathbb{Z} to R/ frak{m}$ and let $S := i( mathbb{Z}) = {z + m mid z in mathbb{Z} }$. Note that "$S$" is a ring. Since $mathbb{Z}[a_1,...,a_n] = { f(a_1,...,a_n) mid f in mathbb{Z}[x_1,...,x_n] }$, it is easy to see that $R/frak{m} = mathbb{Z}[a_1,...,a_n] / frak{m} =$ $S[bar a_1, ..., bar a_n]$, where $bar a_i = a_i + frak{m}$, is a finitely generated $S$-algebra, hence a finite degree extension of the quotient field of $S$ (according to Zariski's lemma).
                    Let's assume that $i$ is injective $Longleftrightarrow mathbb{Z} = S$.
                    Thus $R/ frak{m}$ is a finite degree extension of $mathbb{Q}$. From this we conclude that the elements $bar a_1,..., bar a_n$ with $R/ mathfrak{m} = mathbb{Z}[bar a_1,..., bar a_n]$ are algebraic over $mathbb{Q}$ which means that they are zeros of certain monic polynomials with coefficients in $mathbb{Q}$. Now let $k$ be the product of the denominators of the coefficients of the minimal polynomials of the $bar a_i$ over $mathbb{Q}$. As a consequence, the coefficients of the minimal polynomials can be considered as elements of $mathbb{Z}[ frac{1}{k}]$, hence the $bar a_i$ are integral over $mathbb{Z}[ frac{1}{k}]$ and so the whole $R/ mathfrak{m}$ is integral over $mathbb{Z}[ frac{1}{k}]$.



                    Let $a in mathbb{Z}[ frac{1}{k}], a neq 0$ arbitrary, thus $a in mathbb{Q} subseteq R/ frak{m}$. Since $R/ frak{m}$ is (a field and) integral over $mathbb{Z}[ frac{1}{k}]$, there exist $m in mathbb{N}, c_0,...,c_{m-1} in mathbb{Z}[ frac{1}{k}]$ such that
                    $$a^{-m} + c_{m-1} a^{-(m-1)}+cdots+c_0 = 0,$$ and thus
                    $$a^{-1}=-(c_{n-1}+cdots+c_0 a^{m-1}) in mathbb{Z}[ frac{1}{k}].$$
                    Hence $mathbb{Z}[ frac{1}{k}]$ is a field, but that's a contradiction, since any prime number $q$, which does not divide $k$, cannot be invertible in $mathbb{Z}[ frac{1}{k}]$, because there is no element like $frac{1}{q}$. So $i$ cannot be injective.



                    Thus there exists a $p in mathbb{N}$ prime such that $S = mathbb{Z}/ p mathbb{Z}$, hence $S$ is a finite field. From above we conclude that $R/ frak{m}$ is a finite degree extension of a finite field, thus a finite field. $Box$







                    share|cite|improve this answer














                    share|cite|improve this answer



                    share|cite|improve this answer








                    edited Aug 27 '16 at 9:43









                    user26857

                    39.3k124183




                    39.3k124183










                    answered Aug 27 '16 at 9:33









                    AlgebrusAlgebrus

                    434211




                    434211























                        2












                        $begingroup$

                        This is a mild generalization of Justin Campbell's answer, and the proof is almost the same as that of Proposition 7.8 in Atiyah and MacDonald's Introduction to Commutative Algebra. (This proposition is sometimes called the Artin-Tate Lemma.) The notation has been designed to make the analogy as clear as possible.




                        Let $Asubset Bsubset C$ be a tower of domains such that $B$ is the field of fractions of $A$, and $C$ is a field which is finitely generated as an $A$-algebra. Then (a) $C$ is a finite extension of $B$, and (b) $B$ is a finitely generated $A$-algebra.




                        As (a) results immediately from Zariski's Lemma, it suffices to prove (b). The following statement is ironically stronger and easier to prove than (b):




                        Assume that $C=A[x_1,dots,x_m]$; that $y_1,dots,y_n$ is a $B$-basis of $C$ with $y_1=1$; and that we have $$x_i=sum_jb_{ij}y_j,quad y_iy_j=sum_kb_{ijk}y_k$$ with $b_{ij}$, $b_{ijk}$ in $B$. Then the $A$-algebra $B$ is generated by the $b_{ij}$ and the $b_{ijk}$.




                        The proof is straightforward, and left to the reader.






                        share|cite|improve this answer









                        $endgroup$


















                          2












                          $begingroup$

                          This is a mild generalization of Justin Campbell's answer, and the proof is almost the same as that of Proposition 7.8 in Atiyah and MacDonald's Introduction to Commutative Algebra. (This proposition is sometimes called the Artin-Tate Lemma.) The notation has been designed to make the analogy as clear as possible.




                          Let $Asubset Bsubset C$ be a tower of domains such that $B$ is the field of fractions of $A$, and $C$ is a field which is finitely generated as an $A$-algebra. Then (a) $C$ is a finite extension of $B$, and (b) $B$ is a finitely generated $A$-algebra.




                          As (a) results immediately from Zariski's Lemma, it suffices to prove (b). The following statement is ironically stronger and easier to prove than (b):




                          Assume that $C=A[x_1,dots,x_m]$; that $y_1,dots,y_n$ is a $B$-basis of $C$ with $y_1=1$; and that we have $$x_i=sum_jb_{ij}y_j,quad y_iy_j=sum_kb_{ijk}y_k$$ with $b_{ij}$, $b_{ijk}$ in $B$. Then the $A$-algebra $B$ is generated by the $b_{ij}$ and the $b_{ijk}$.




                          The proof is straightforward, and left to the reader.






                          share|cite|improve this answer









                          $endgroup$
















                            2












                            2








                            2





                            $begingroup$

                            This is a mild generalization of Justin Campbell's answer, and the proof is almost the same as that of Proposition 7.8 in Atiyah and MacDonald's Introduction to Commutative Algebra. (This proposition is sometimes called the Artin-Tate Lemma.) The notation has been designed to make the analogy as clear as possible.




                            Let $Asubset Bsubset C$ be a tower of domains such that $B$ is the field of fractions of $A$, and $C$ is a field which is finitely generated as an $A$-algebra. Then (a) $C$ is a finite extension of $B$, and (b) $B$ is a finitely generated $A$-algebra.




                            As (a) results immediately from Zariski's Lemma, it suffices to prove (b). The following statement is ironically stronger and easier to prove than (b):




                            Assume that $C=A[x_1,dots,x_m]$; that $y_1,dots,y_n$ is a $B$-basis of $C$ with $y_1=1$; and that we have $$x_i=sum_jb_{ij}y_j,quad y_iy_j=sum_kb_{ijk}y_k$$ with $b_{ij}$, $b_{ijk}$ in $B$. Then the $A$-algebra $B$ is generated by the $b_{ij}$ and the $b_{ijk}$.




                            The proof is straightforward, and left to the reader.






                            share|cite|improve this answer









                            $endgroup$



                            This is a mild generalization of Justin Campbell's answer, and the proof is almost the same as that of Proposition 7.8 in Atiyah and MacDonald's Introduction to Commutative Algebra. (This proposition is sometimes called the Artin-Tate Lemma.) The notation has been designed to make the analogy as clear as possible.




                            Let $Asubset Bsubset C$ be a tower of domains such that $B$ is the field of fractions of $A$, and $C$ is a field which is finitely generated as an $A$-algebra. Then (a) $C$ is a finite extension of $B$, and (b) $B$ is a finitely generated $A$-algebra.




                            As (a) results immediately from Zariski's Lemma, it suffices to prove (b). The following statement is ironically stronger and easier to prove than (b):




                            Assume that $C=A[x_1,dots,x_m]$; that $y_1,dots,y_n$ is a $B$-basis of $C$ with $y_1=1$; and that we have $$x_i=sum_jb_{ij}y_j,quad y_iy_j=sum_kb_{ijk}y_k$$ with $b_{ij}$, $b_{ijk}$ in $B$. Then the $A$-algebra $B$ is generated by the $b_{ij}$ and the $b_{ijk}$.




                            The proof is straightforward, and left to the reader.







                            share|cite|improve this answer












                            share|cite|improve this answer



                            share|cite|improve this answer










                            answered May 19 '17 at 10:57









                            Pierre-Yves GaillardPierre-Yves Gaillard

                            13.4k23184




                            13.4k23184






























                                draft saved

                                draft discarded




















































                                Thanks for contributing an answer to Mathematics Stack Exchange!


                                • Please be sure to answer the question. Provide details and share your research!

                                But avoid



                                • Asking for help, clarification, or responding to other answers.

                                • Making statements based on opinion; back them up with references or personal experience.


                                Use MathJax to format equations. MathJax reference.


                                To learn more, see our tips on writing great answers.




                                draft saved


                                draft discarded














                                StackExchange.ready(
                                function () {
                                StackExchange.openid.initPostLogin('.new-post-login', 'https%3a%2f%2fmath.stackexchange.com%2fquestions%2f148745%2ffields-finitely-generated-as-mathbb-z-algebras-are-finite%23new-answer', 'question_page');
                                }
                                );

                                Post as a guest















                                Required, but never shown





















































                                Required, but never shown














                                Required, but never shown












                                Required, but never shown







                                Required, but never shown

































                                Required, but never shown














                                Required, but never shown












                                Required, but never shown







                                Required, but never shown







                                Popular posts from this blog

                                Biblatex bibliography style without URLs when DOI exists (in Overleaf with Zotero bibliography)

                                ComboBox Display Member on multiple fields

                                Is it possible to collect Nectar points via Trainline?